PT 33 Section 4 Game 2 Forum

Prepare for the LSAT or discuss it with others in this forum.
Post Reply
dadownclub8

New
Posts: 53
Joined: Wed May 13, 2009 6:35 pm

PT 33 Section 4 Game 2

Post by dadownclub8 » Thu Oct 10, 2013 3:52 pm

Hi. I'm having a bit of trouble with Q8.

Part of the Logic chain I have is the contrapositive which is J or M->H. They tell you that J is not in the forest. Correct me if I'm wrong but or is inclusive. I keep wanting to think that since OR is inclusive these are the three possibilities if I break down the aforementioned chain.

J->H
M->H
Or both J and M are there to produce H.

Can someone explain to me whether or not, if this is wrong and if so why it is wrong?

I chose answer choice E because based upon those possibilities, why is it that if you know that you don't have J, that M wouldn't be there for certain? Given the three possibilities, it seems inherent that M would be there for certain? Maybe I'm not understanding. I apologize if this is a ridiculously stupid question but it's been bugging me.
Last edited by dadownclub8 on Thu Oct 10, 2013 4:03 pm, edited 1 time in total.

User avatar
toshiroh

Bronze
Posts: 438
Joined: Wed Oct 10, 2012 1:58 pm

Re: PT 33 Section 4 Game 2

Post by toshiroh » Thu Oct 10, 2013 3:59 pm

If you go to the Manhattan LSAT website, you'll probably have a better chance of getting your question answered. I'm pretty sure someone has already asked it.

dosto

Silver
Posts: 784
Joined: Fri Oct 04, 2013 12:50 am

Re: PT 33 Section 4 Game 2

Post by dosto » Thu Oct 10, 2013 4:07 pm

.
Last edited by dosto on Fri Sep 25, 2015 3:04 am, edited 1 time in total.

Cambridge LSAT

Bronze
Posts: 257
Joined: Mon Aug 24, 2009 3:26 pm

Re: PT 33 Section 4 Game 2

Post by Cambridge LSAT » Thu Oct 10, 2013 4:23 pm

The rule allows four options:
  • In: J H; Out: M
  • In: M H; Out: J
  • In: H J M
  • Out: H J M
It sounds like you forgot that all three can be out.

User avatar
AAJD2B

Silver
Posts: 871
Joined: Sat Dec 01, 2012 12:37 am

Re: PT 33 Section 4 Game 2

Post by AAJD2B » Thu Oct 10, 2013 4:42 pm

You're overlooking one of the rules that states if J is not in, then S is in. The correct answer choice states S is not in.

E is a could be true scenario and is not a must be false. We can have H and S as the only variables and everything else out.

Want to continue reading?

Register now to search topics and post comments!

Absolutely FREE!


dadownclub8

New
Posts: 53
Joined: Wed May 13, 2009 6:35 pm

Re: PT 33 Section 4 Game 2

Post by dadownclub8 » Fri Oct 11, 2013 2:54 pm

So just to make sure I understand it properly. This is more about the logic behind this part of the Q.

If the scenario is J or M->H
and you know ~J
therefore there's nothing you can logically conclude from that correct? This is because you don't know anything about M and since you don't know about M, H is up in the air as well correct? So it could be true, but not necessarily true.

dosto

Silver
Posts: 784
Joined: Fri Oct 04, 2013 12:50 am

Re: PT 33 Section 4 Game 2

Post by dosto » Fri Oct 11, 2013 3:00 pm

.

Want to continue reading?

Register for access!

Did I mention it was FREE ?


Post Reply

Return to “LSAT Prep and Discussion Forum”